Answer:
y = 1.6x
Step-by-step explanation:
y is the total amount/pounds of almonds that Lark bought. x is the amount that Tessa bought. 3/5 is the same as 0.6. If we multiplied Tessa's amount by 0.6 it would decrease by 2/5 so we have to add 1 to make it 1.6. To find how much Lark bought multiply 1.6 and Tessa's amount.
Answer:
how many 35 (x) did molly purchase, for 5 (y) weeks.
Step-by-step explanation:
I chose this because each number represents a different volume , it also makes a good answer, comment or repsond if i am wrong.
A firm makes circular drink mats , of radius 4.5cm , that are 3mm thick. they want to produce a rectangular box to hold a pile of 12 mats. what are the minium dimensions of the box.
The minimum dimensions of the rectangular box is 90mm by 90mm by 36mm when converted to millimeter
Dimensions of a rectangular boxThere are three dimensions of a rectangular box which are the length, width, and height . The three measurements are multiplied together using Length × Width × Height in the same units.
The radius of the circular mat = 4.5cm and 45{converted to millimeter}
the total length of the mat = 2 × 45mm {2radius = diameter}
the total length of the mat = 90mm
Since the length of the circular mat is the same around the mat, hence the rectangular box has two dimensions of length of 90mm and width of 90mm
The total thickness of the 12 mats will determine the height of the rectangular box so;
the thickness of a mat is given as 3mm
total thickness of 12 mats = 3mm × 12
total thickness of 12 mats = 36mm
Therefore, the rectangular box will have a minimum dimension of length 90mm by width 90mm by height 36mm
Know more about dimension of rectangular box https://brainly.ph/question/11621807
#SPJ1
Helppppppppppp!!!!! Please
Answer:
The answer is C.
Step-by-step explanation:
angle 5 and 4 makes a line which is 180 and since angle 5 equals 57 , 180-57=123.
Simplify (3x − 5) + (3x + 6)
Answer:
6x+1
Step-by-step explanation:
3x+3x=6x
-5+6=1
Answer:
6x+1
Step-by-step explanation:
First remove the parentheses
the combine like terms!
6x+1
Hoped it help!
825 use each digit once. make the smallest 3digit number
Step-by-step explanation:
Given: To make smallest 3-digit number of 825.
To find: The smallest 3-digit number of 825.
Solution: We can make the smallest 3-digit number of 825 by separating the numbers and arranging it to ascending order. The given number is 825. ...
Final answer: The smallest 3-digit number of 825 is 258.
hope it helps
Answer:
258
Step-by-step explanation:
We are given 3 numbers:
8 2 5
And we are asked to find the smallest 3 digit number using those 3 digits above.
To make the smallest number, place the numbers in value from least to greatest:
2 5 8
This is your 3 digit number: 258.
Hope this helps! :)
Work out the volume of the prism height of 12 4 and five
The calculated volume of the prism is 702 cubic cm
Finding the volume of the prismFrom the question, we have the following parameters that can be used in our computation
The trapezoidal prism (see attachment)
The formula of the volume of a trapezoidal prism is
Volume = Base area * Height
Where we have
Base area = 1/2 * (8 + 10) * 6
Evaluate the sum of 8 and 10
Base area = 1/2 * 18 * 6
Evaluate the products of 1/2, 18 and 6
Base area = 54
Also, we have
Height = 13
So, the volume is calculated as
volume = 13 * 54
Evaluate
volume = 702
Hence, the volume of the prism is 702 cubic cm
Read more about volume at
brainly.com/question/463363
#SPJ1
A distribution has a mean of 90 and a standard deviation of 15. Samples of size 25 are drawn randomly from the population. Find the probability that the sample mean is more than 85 g
Answer:
The probability is 0.04746
Step-by-step explanation:
Firstly, we calculate the z-score here
Mathematically;
z-score = x-mean/SD/√n
Where from the question;
x = 85, mean = 90 , SD = 15 and n = 25
Plugging these values into the equation, we have;
Z = (85-90)/15/√25 = -5/15/5 = -1.67
So the probability we want to calculate is ;
P(z > -1.67)
We use the standard normal distribution table for this;
P(z > -1.67) = 0.04746
Using the diagram, find the measure of the angle indicated in bold.
A 95
B 110
C 120
D 130
what is the answered
pp lssss answer
\(\boxed{ \underline{ \bf \: Categorical}} \sf \: data \: set \: is \: the \: data \: set \\ \sf \: that \: deals \: with \: only \: non \: numerical \: values.\)
Optoion C. Categorial data set is the right answer of the question given.
RaonbowSalt2222
Find the height of the tower using the information given in the illustration.
using SOH CAH TOA
Tan 85.144 =h/130
h=tan 85.144*130
h=1530.19 fr
Pleaseee for the love of god help
Answer:
The Radius is the distance from the center outwards. The Diameter goes straight across the circle, through the center. The Circumference is the distance once around the circle.
The graph of which equation is shown?
Answer:
it should be
C. y = 2x +2
Step-by-step explanation:
Answer:
A
Step-by-step explanation:
The starting point is 2. Where the line touches the y axis. The line as you see is going down which is negative. Pick 2 points on the graph where it’s touching. It is going down by 2. Hope this helps!
find the vertex of the graph of the following quadratic equations y= -3x^2 -12x-13
Answer:
Step-by-step explanation:
The vertex is a point. The x value is defined as x = - b / 2a where
a = -3
b = - 12
x = - -12/ 2* -3
x = 12 / - 6
x = -2
what is y when x = - 2?
y = -3(-2)^2 - 12(-2) - 13
y = -12 + 24 - 13
y = -1
Does the graph agree? It should
The equation is written as
y = -3(x + 2)^2 - 1
hey yall what is 34 x 45 x 567
Answer:
867510
Step-by-step explanation:
34 x 45 = 1530
1530 x 567 = 867510
hope this helped
The answer is
\(867510\)
-5a - 2.2 <17.8? Solve for a
Answer:
To solve for the value of a, you have to isolate “a“ on one side.
-5a - 2.2 < 17.8
Add 2.2 to both sides.
-5a < 20
Divide both sides by -5. Note: when you are dividing by a negative number, the direction of the sign switches.
a > -4
:)
Help me please and thank you, and I will mark you brainliest!!
Answer:
270m^2
Step-by-step explanation:
30m*18m/2 = 270m^2
Answer:
270
Step-by-step explanation:
A clothing business finds there is a linear relationship between the number of shirts, n , it can sell and the price, p , it can charge per shirt. In particular, historical data shows that 1,000 shirts can be sold at a price of $30 , while 3,000 shirts can be sold at a price of $22 . Find a linear equation in the form p(n)=mn+b that gives the price p they can charge for n shirts.
The linear equation in the form p(n) = mn + b that gives the price p they can charge for n shirt is p(n) = - 1 / 250n + 34.
How to linear equation in slope intercept form?Linear equation can be represented in slope intercept form as follows:
y = mx + b
where
m = slopeb = y-interceptTherefore, the clothing business finds there is a linear relationship between the number of shirts, n, it can sell and the price, p , it can charge per shirt. 1,000 shirts can be sold at a price of $30 , while 3,000 shirts can be sold at a price of $22 .
Therefore,
using (1000, 30) and (3000, 22) let's find the slope.
m = 22 - 30 / 3000 - 1000
m = - 8 / 2000
m = - 1 / 250
Hence, let's find the y-intercept using (1000, 30)
p(n) = mn + b
where
p = pricen = number of shirtTherefore,
30 = - 1 / 250 (1000) + b
30 = - 4 + b
b = 30 + 4
b = 34
Hence, the linear equation is p(n) = - 1 / 250n + 34
learn more on linear equation here: https://brainly.com/question/14495872
#SPJ1
Two hundred eighty-two people attended a recent performance of Cinderella. Adult tickets sold for $5 and children’s tickets sold for $3 each. Find the number of adults and the number of children that attended the play if the total revenue was $1046.
Part A: Write a system of equations in standard form (Ax + By = C) that can be solved to find the number of adults and children who attended the performance. Define the variables used in the equations. (4 points)
Part B: How many adults attended the performance? How many children attended the performance? Show your work and steps of how you found your answer using elimination.
A. A system of equations in standard form that can be solved to find the number of adults and children who attended the performance is:
x + y = 282
3x + 5y = 1046
B. The number of adults who attended the performance is 182 adults.
The number of children who attended the performance is 100 children.
How to determine the number of each type of tickets sold?In order to write a system of linear equations to describe this situation, we would assign variables to the number of adult tickets sold and number of children tickets sold, and then translate the word problem into an algebraic equation as follows:
Let the variable x represent the number of adult tickets sold.Let the variable y represent the number of children tickets sold.Since 282 people attended the recent performance by Cinderella, a linear equation that models the situation is given by:
x + y = 282 ....equation 1.
Additionally, adult tickets sold for $5 while children tickets sold for $3 each with a total revenue was $1046, a linear equation that models the situation is given by:
3x + 5y = 1046 .......equation 2.
Part B.
By multiplying equation 1 by 3, we have:
3x + 3y = 846 .......equation 3.
By subtracting equation 3 from equation 2, we have:
2y = 200
y = 100 children.
For the x-value, we have:
x = 282 - y
x = 282 - 100
x = 182 adults.
Read more on solution and equation here: brainly.com/question/25858757
#SPJ1
Five boxes of fruit snacks cost $22.50.
How much would three boxes cost?
How much would nine boxes cost?
How much would x boxes cost?
Answer:
13.5 answer is 13.5 and one cost 4.5
Let f be a function defined on the set of positive rational numbers with the property that f(a · b) = f(a) + f(b) for all positive rational numbers a and b. Suppose that f also has the property that f(p) = p for every prime number p. For which of the following numbers x is f(x) < 0?
a. 17/32
b. 11/16
c. 7/9
d. 7/6
e. 25/11
The figure shown is created by joining two rectangles. What is the area in square inches of the figure?
Answer:
The answer is 39 square inches.
Four tiles lettered H, O, P, and E are face down on a table. A tile is selected, the letter recorded, the tile replaced, and the process is repeated. What is the experimental probability of selecting an H? The results are H-5, O-5, P-4, E-6.
Answer:
1/4
Step-by-step explanation:
Given that :
Result obtained : H-5, O-5, P-4, E-6
Hence, number of trials = (5 + 5 + 4 + 6) = 20
Experimental probability :
(Number of occurences of an event / total number of trials)
Experimental probability of selecting an H;
Number of occurrences of H = 5
Total number of trials = 20
P(selecting an H) = 5/20
= 1/4
Hence, the experimental probability of selecting an H is 1/4
Domain and range and function
Choices for domain are
X | X greater than 0 or
X | x greater than or equal to 0 or
All real numbers
Square bracket 0 infinity
Parenthesis negative infinity positive infinity
Range
Y | Y greater than 0 or
Y | Y greater than or equal to 0 or
All real numbers
Square bracket 0 infinity or
Parenthesis 0 infinity or
negative infinity positive infinity
Function 1 to 1 , onto both 1 to 1 and onto
Or neither 1 to 1 nor onto
And discrete , continuous or neither continuous nor discrete
#1
Domain
The set of x valuesHere the arrows are going infinite directions
(-oo,oo)All real numbers#2
Smae for range
The set of y valuesArrows approaching infinity
(-oo,oo)All real numbers#3
It fails in horizontal line test and vertical line test
One one and ontoAlso
The function is continuous everywhereSketch and shade the region in the xy-plane defined by the equation or inequalities.
|x| < 7 and |y| < 3 g
Answer:
attached below is the solution
Step-by-step explanation:
|x| < 7
= -7 < x < 7
| y | < 3
= -3< y < 3
attached below is the shaded region in the xy-plane
Books are on sale for $7. Peter has $30 dollars in his wallet. How many books can he buy?( interpret the remainder)
Answer:
He can buy 4 books with 2 dollars remaining
Step-by-step explanation:
30/7 = 4 R2
Answer: 4 books.
Explanation: First you divide $30 by 7, and you get 4 and 2/7 (2/7 as the remainder) left over. So you can't have 4 and 2/7 books, it has to be a whole number.
Therefore, he can buy 4 books. I hope this helped!
Antonio wants to examine the sales in a restaurant during a period of time from 2 hours before to 2 hours after the peak dinner time and graph his results. If the peak dinner time is considered to be at 6 p.m., what would be the domain for the graph Antonio is creating?
Using function concepts, it is found that the domain of the graph Antonio is creating would be: \(4 \leq x \leq 8\)
The domain of a function is the set that contains all possible input values.In this problem, the inputs are the time, between 2 hours before and 2 hours after the peak time of 6 pm, hence, it is given by: \(4 \leq x \leq 8\)
For more on the domain of functions, you can check https://brainly.com/question/24374080
complement of 0.7253
Answer:
Step-by-step explanation:
if it is converting its 7253/10000
to percent 72.53
scientific notation is 7.253 *10-1 the -1 is on top of the 10
PLEASE HELP
Suppose that the functions fand g are defined for all real numbers x as follows.
f(x) = 5x
g(x)=4x-4
Write the expressions for (g.f)(x) and (g-f)(x) and evaluate (g+f)(2).
(g•f)(x) =
(g-f)(x) =
(g+r) (2)=
X=Y+Y xy So, x=X-Y/Yy True False
The statement X = Y + Yxy is True.
The equation given is:
X = Y + Yxy
To find x = (X- Y) / Yy we have to substitute the value of x in X = Y + Yxy as
X = Y + Yxy
X = Y + Yy (X-Y) / Yy
To solve for x, we can rearrange the equation as:
X = Y + X - Y
X = X
Thus, the statement is True.
Learn more about Equation here:
https://brainly.com/question/29657983
#SPJ1
Can someone help me?
The midpoint of the fourth class interval is 0.51
The range of the fourth class is 0.48-0.54
So, to find the middle point of the class we use the following formula:
The average of the top and lower values of a specific class interval is referred to as the class midpoint (also known as the class mark). The class midpoint substitutes a range of values with a single value in a frequency distribution table. We can create a line graph for our data using the frequency and the class midpoint.
Midpoint = Upper limit+Lower limit/2
Upper limit = 0.54
Lower limit = 0.48
Substituting the values in the formula we get:
Midpoint = (0.54+0.48)/2
= 0.51
Learn more about class intervals:
https://brainly.com/question/19473137
#SPJ1
square root of the quantity x plus 7 end quantity minus 1 equals x
Answer:
x = 2
Step-by-step explanation:
√(x + 7) − 1 = x
√(x + 7) = x + 1
x + 7 = (x + 1)²
x + 7 = x² + 2x + 1
0 = x² + x − 6
0 = (x + 3) (x − 2)
x = -3 or 2
Check for extraneous solutions:
√(-3 + 7) − 1 = -3
√4 − 1 = -3
2 − 1 = -3
1 = -3
x ≠ -3
√(2 + 7) − 1 = 2
√9 − 1 = 2
3 − 1 = 2
2 = 2
x = 2